Consider a box with width w length l and height h suppose


Consider a box with width W, length L, and height H. Suppose you have to construct a box with the following constraint: W+L+H equal or less than 30.

a) Suppose W has to be 12. What is maximizing volume of box? What should L and H be?

b) Suppose W has to be some number x element of set (0,30). In order maximize volume of the box, what should L and H be?

c) Using answers from part B, in order to maximize the box what is the W, L, H?

Request for Solution File

Ask an Expert for Answer!!
Business Economics: Consider a box with width w length l and height h suppose
Reference No:- TGS0986629

Expected delivery within 24 Hours